This commit is contained in:
Josia Pietsch 2024-02-07 16:03:42 +01:00
commit ffcd9cd45f
Signed by: josia
GPG Key ID: E70B571D66986A2D
20 changed files with 291 additions and 188 deletions

View File

@ -44,4 +44,12 @@ title = {Classical Descriptive Set Theory},
volume = {156}, volume = {156},
year = {2012}, year = {2012},
} }
@MISC{3722713,
TITLE = {Embedding of countable linear orders into $\Bbb Q$ as topological spaces},
AUTHOR = {Eric Wofsey},
HOWPUBLISHED = {Mathematics Stack Exchange},
NOTE = {URL:https://math.stackexchange.com/q/3722713 (version: 2020-06-16)},
EPRINT = {https://math.stackexchange.com/q/3722713},
URL = {https://math.stackexchange.com/q/3722713}
}

View File

@ -164,8 +164,10 @@
\] \]
\end{notation} \end{notation}
\gist{%
The following similar to Fubini, The following similar to Fubini,
but for meager sets: but for meager sets:
}{}
\begin{theorem}[Kuratowski-Ulam] \begin{theorem}[Kuratowski-Ulam]
\yalabel{Kuratowski-Ulam}{Kuratowski-Ulam}{thm:kuratowskiulam} \yalabel{Kuratowski-Ulam}{Kuratowski-Ulam}{thm:kuratowskiulam}
@ -193,6 +195,7 @@ but for meager sets:
\end{enumerate} \end{enumerate}
\end{theorem} \end{theorem}
\begin{refproof}{thm:kuratowskiulam} \begin{refproof}{thm:kuratowskiulam}
\gist{
(ii) and (iii) are equivalent by passing to the complement. (ii) and (iii) are equivalent by passing to the complement.
\begin{claim}%[1a] \begin{claim}%[1a]
@ -286,16 +289,11 @@ but for meager sets:
$M_x$ is comeager $M_x$ is comeager
as a countable intersection of comeager sets. as a countable intersection of comeager sets.
\end{refproof} \end{refproof}
}{}
% \phantom\qedhere % \phantom\qedhere
% \end{refproof} % \end{refproof}
% TODO fix claim numbers % TODO fix claim numbers
\gist{%
\begin{remark}
Suppose that $A$ has the BP.
Then there is an open $U$ such that
$A \symdif U \mathbin{\text{\reflectbox{$\coloneqq$}}} M$ is meager.
Then $A = U \symdif M$.
\end{remark}
}{}

View File

@ -1,8 +1,8 @@
\lecture{06}{2023-11-03}{} \lecture{06}{2023-11-03}{}
\gist{%
% \begin{refproof}{thm:kuratowskiulam} % \begin{refproof}{thm:kuratowskiulam}
\begin{enumerate}[(i)] \begin{enumerate}[(i)]
\item Let $A$ be a set with the Baire Property. \item Let $A$ be a set with the Baire property.
Write $A = U \symdif M$ Write $A = U \symdif M$
for $U$ open and $M$ meager. for $U$ open and $M$ meager.
Then for all $x$, Then for all $x$,
@ -51,8 +51,8 @@
Towards a contradiction suppose that $A$ is not meager. Towards a contradiction suppose that $A$ is not meager.
Then $U$ is not meager. Then $U$ is not meager.
Since $X \times Y$ is second countable, Since $X \times Y$ is second countable,
we have that $A$ is a countable union of open rectangles. we have that $U$ is a countable union of open rectangles.
At least one of them, say $G \times H \subseteq A$, At least one of them, say $G \times H \subseteq U$,
is not meager. is not meager.
By \yaref{thm:kuratowskiulam:c2}, By \yaref{thm:kuratowskiulam:c2},
both $G$ and $H$ are not meager. both $G$ and $H$ are not meager.
@ -71,7 +71,59 @@
``$\implies$'' ``$\implies$''
This is \yaref{thm:kuratowskiulam:c1b}. This is \yaref{thm:kuratowskiulam:c1b}.
\end{enumerate} \end{enumerate}
}{%
\begin{itemize}
\item (ii) $\iff$ (iii): pass to complement.
\item $F \overset{\text{closed}}{\subseteq} X \times Y$ nwd.
$\implies \{x \in X : F_x \text{ nwd}\} $ comeager:
\begin{itemize}
\item $W = F^c$ is open and dense, show that $\{x : W_x \text{ dense}\}$
is comeager.
\item $(V_n)$ enumeration of basis. Show that $U_n \coloneqq \{x : V_n \cap W_x \neq \emptyset\}$
is comeager for all $n$.
\item $U_n$ is open (projection of open) and dense ($W$ is dense, hence $W \cap ( U \times V_n) \neq \emptyset$ for $U$ open).
\end{itemize}
\item $F \subseteq X \times Y$ is nwd $\implies \{x \in X: F_x \text{ nwd}\}$ comeager.
(consider $\overline{F}$).
\item (ii) $\implies$:
$M \subseteq X \times Y$ meager $\implies \{x \in X: M_x \text{ meager}\}$ comeager
(write $M$ as ctbl. union of nwd.)
\item (i): If $A$ has the Baire Property,
then $A = U \symdif M$, $A_x = U_x \symdif M_x$,
$U_x$ open and $\{x : M_x \text{ meager}\}$ comeager
$\implies$ (i).
\item $P \subseteq X$, $Q \subseteq Y$ BP,
then $P \times Q$ meager $\iff$ $P$ or $Q$ meager.
\begin{itemize}
\item $\impliedby$ easy
\item $\implies$ Suppose $P \times Q$ meager, $P$ not meager.
$\emptyset\neq P \cap \underbrace{\{x : (P \times Q)_x \text{ meager} \}}_{\text{comeager}} \ni x$.
$(P \times Q)_x = Q$ is meager.
\end{itemize}
\item (ii) $\impliedby$:
\begin{itemize}
\item $A$ BP, $\{x : A_x \text{ meager}\}$ comeager.
\item $A = U \symdif M$.
\item Suppose $A$ not meager $\leadsto$ $U$ not meager
$\leadsto \exists G \times H \subseteq U$ not meager.
\item $G$ and $H$ are not meager.
\item $\exists x_0 \in G \cap \underbrace{\{x: A_x \text{ meager } \land M_x \text{ meager}\}}_\text{comeager}$.
\item $H$ meager, as
\[
H \subseteq U_{x_0} \subseteq A_{x_0} \cup M_{x_0}.
\]
\end{itemize}
\end{itemize}
}
\end{refproof} \end{refproof}
\gist{%
\begin{remark}
Suppose that $A$ has the BP.
Then there is an open $U$ such that
$A \symdif U \mathbin{\text{\reflectbox{$\coloneqq$}}} M$ is meager.
Then $A = U \symdif M$.
\end{remark}
}{}
\section{Borel sets} % TODO: fix chapters \section{Borel sets} % TODO: fix chapters

View File

@ -18,20 +18,10 @@ by associating a function $f\colon \Q \to \{0,1\}$
with $(f^{-1}(\{1\}), <)$. with $(f^{-1}(\{1\}), <)$.
\begin{lemma} \begin{lemma}
Any countable ordinal embeds into $(\Q,<)$. Any countable wellorder embeds into $(\Q,<)$.
\end{lemma} \end{lemma}
\begin{proof}[sketch] \begin{proof}\footnote{In the lecture this was only done for countable \emph{ordinals}.}
Use transfinite induction. Cf.~\cite{3722713}.
Suppose we already have $\alpha \hookrightarrow (\Q, <)$,
we need to show that $\alpha +1 \hookrightarrow (\Q, <)$.
Since $(0,1) \cap \Q \cong \Q$,
we may assume $\alpha \hookrightarrow ((0,1), <)$
and can just set $\alpha \mapsto 2$.
For a limit $\alpha$
take a countable cofinal subsequence $\alpha_1 < \alpha_2 < \ldots \to \alpha$.
Then map $[0,\alpha_1)$ to $(0,1)$
and $[\alpha_i, \alpha_{i+1})$ to $(i,i+1)$.
\end{proof} \end{proof}
% TODO $\WF \subseteq 2^\Q$ is $\Sigma^1_1$-complete. % TODO $\WF \subseteq 2^\Q$ is $\Sigma^1_1$-complete.

View File

@ -273,6 +273,7 @@ Recall:
A limit of distal flows is distal. A limit of distal flows is distal.
\end{proposition} \end{proposition}
\begin{proof} \begin{proof}
\gist{%
Let $(X,T)$ be a limit of $\Sigma = \{(X_i, T) : i \in I\}$. Let $(X,T)$ be a limit of $\Sigma = \{(X_i, T) : i \in I\}$.
Suppose that each $(X_i, T)$ is distal. Suppose that each $(X_i, T)$ is distal.
If $(X,T)$ was not distal, If $(X,T)$ was not distal,
@ -283,4 +284,7 @@ Recall:
But then $g_n \pi_i(x_1) \to \pi_i(z)$ But then $g_n \pi_i(x_1) \to \pi_i(z)$
and $g_n \pi_i(x_2) \to \pi_i(z)$, and $g_n \pi_i(x_2) \to \pi_i(z)$,
which is a contradiction since $(X_i, T)$ is distal. which is a contradiction since $(X_i, T)$ is distal.
}{Suppose there is a proximal pair $x_1,x_2$.
Take $i$ such that $\pi_i(x_1) \neq \pi_i(x_2) \lightning$.
}
\end{proof} \end{proof}

View File

@ -17,13 +17,13 @@ $X$ is always compact metrizable.
% and $h\colon x \mapsto x + \alpha$.\footnote{Again $x + \alpha$ denotes $x \cdot \alpha$ in $\C$.} % and $h\colon x \mapsto x + \alpha$.\footnote{Again $x + \alpha$ denotes $x \cdot \alpha$ in $\C$.}
% \end{example} % \end{example}
\begin{proof} \begin{proof}
% TODO TODO TODO Think! \gist{%
The action of $1$ determines $h$. The action of $1$ determines $h$.
Consider Consider
\[ \[
\{h^n : n \in \Z\} \subseteq \cC(X,X)\gist{ = \{f\colon X \to X : f \text{ continuous}\}}{}, \{h^n : n \in \Z\} \subseteq \cC(X,X)\gist{ = \{f\colon X \to X : f \text{ continuous}\}}{},
\] \]
where the topology is the uniform convergence topology. % TODO REF EXERCISE where the topology is the uniform convergence topology.
Let $G = \overline{ \{h^n : n \in \Z\} } \subseteq \cC(X,X)$. Let $G = \overline{ \{h^n : n \in \Z\} } \subseteq \cC(X,X)$.
Since the family $\{h^n : n \in \Z\}$ is uniformly equicontinuous, Since the family $\{h^n : n \in \Z\}$ is uniformly equicontinuous,
i.e.~ i.e.~
@ -82,6 +82,15 @@ $X$ is always compact metrizable.
For $\alpha = h$ we get that For $\alpha = h$ we get that
a flow $\Z \acts X$ corresponds to $\Z \acts K$ a flow $\Z \acts X$ corresponds to $\Z \acts K$
with $(1,x) \mapsto x + \alpha$. with $(1,x) \mapsto x + \alpha$.
}{
\begin{itemize}
\item $G \coloneqq \overline{\{h^n : n \in \Z\} } \subseteq \cC(X,X)$.
\item $G$ is compact (Arzela-Ascoli), abelian topological group (closure of ab. top. group)
\item Take any $x \in G$.
\item $Gx$ is compact (since $g \mapsto gx$ is continuous and $G$ is compact)
\item Stabilizer $G_x$ is closed. $K \coloneqq \faktor{G}{G_x}$, $K \to X, fG_x \mapsto f(x)$.
\end{itemize}
}
\end{proof} \end{proof}
\begin{definition} \begin{definition}
Let $(X,T)$ be a flow Let $(X,T)$ be a flow
@ -151,16 +160,18 @@ By Zorn's lemma, this will follow from
& {(Z,T)} & {(Z,T)}
\arrow[from=1-1, to=1-3] \arrow[from=1-1, to=1-3]
\arrow[from=2-2, to=1-3] \arrow[from=2-2, to=1-3]
\arrow["{\text{isometric extension}}"{description}, from=1-1, to=2-2] \arrow["{\text{isometric extension}}"{description}, from=1-1, to=2-2]
\end{tikzcd}\] \end{tikzcd}\]
\end{theorem} \end{theorem}
\yaref{thm:furstenberg} allows us to talk about ranks of distal minimal flows: \yaref{thm:furstenberg} allows us to talk about ranks of distal minimal flows:
\begin{definition} \begin{definition}[{\cite[{}13.1]{Furstenberg}}]
Let $(X, \Z)$ be distal minimal. \label{def:floworder}
Then $\rank((X,\Z)) \coloneqq \min \{\eta : (X, \Z) \cong (X_\eta, \Z)\}$ Let $(X,T)$ be a quasi-isometric flow,
where $(X_{\eta}, \Z)$ is as from the definition of quasi-isometric flows, and let $\eta$ be the smallest ordinal
i.e.~$\rank((X,\Z))$ is the minimal height such such that there exists a quasi-isometric system $\{(X_\xi, T), \xi \le \eta\}$
that a tower as in the definition exists. with $(X,T) = (X_\xi, T)$.
Then $\eta$ is called the \vocab{rank} or \vocab{order} of the flow
and is denoted by $\rank((X,T))$.
\end{definition} \end{definition}
\begin{definition}+ \begin{definition}+

View File

@ -127,7 +127,7 @@ since $X^X$ has these properties.
\begin{lemma}[EllisNumakura] \begin{lemma}[EllisNumakura]
\yalabel{Ellis-Numakura Lemma}{Ellis-Numakura}{lem:ellisnumakura} \yalabel{Ellis-Numakura Lemma}{Ellis-Numakura}{lem:ellisnumakura}
Every compact semigroup Every non-empty compact semigroup
contains an \vocab{idempotent} element, contains an \vocab{idempotent} element,
i.e.~$f$ such that $f^2 = f$. i.e.~$f$ such that $f^2 = f$.
\end{lemma} \end{lemma}

View File

@ -35,20 +35,13 @@ equicontinuity coincide.
By equicontinuity of $T$ we get that $\tilde{d}$ and $d$ By equicontinuity of $T$ we get that $\tilde{d}$ and $d$
induce the same topology on $X$. induce the same topology on $X$.
\end{proof} \end{proof}
\gist{
Recall that we defined the order of a quasi-isometric flow
to be the minimal number of steps required when building the tower
to reach the flow with a quasi-isometric system (cf.~\yaref{thm:l16:3},
\yaref{def:floworder}).
}{}
\begin{question}
What is the minimal number of steps required
when building the tower to reach the flow
as in \yaref{thm:l16:3}?
\end{question}
\begin{definition}[{\cite[{}13.1]{Furstenberg}}]
Let $(X,T)$ be a quasi isometric flow,
and let $\eta$ be the smallest ordinal
such that there exists a quasi-isometric system $\{(X_\xi, T), \xi \le \eta\}$
with $(X,T) = (X_\xi, T)$.
Then $\eta$ is called the \vocab{order} of the flow.
\end{definition}
\begin{theorem}[Maximal isometric factor] \begin{theorem}[Maximal isometric factor]
\label{thm:maxisomfactor} \label{thm:maxisomfactor}
For every flow $(X,T)$ there is a maximal factor $(Y,T)$, $\pi\colon X\to Y$, For every flow $(X,T)$ there is a maximal factor $(Y,T)$, $\pi\colon X\to Y$,
@ -279,9 +272,6 @@ More generally we can show:
In particular, In particular,
$(X'_{\xi+1}, T)$ is a factor of $(X_{\xi+1}, T)$. $(X'_{\xi+1}, T)$ is a factor of $(X_{\xi+1}, T)$.
\end{proof} \end{proof}
% TODO ANKI-MARKER
\begin{example}[{\cite[p. 513]{Furstenberg}}] \begin{example}[{\cite[p. 513]{Furstenberg}}]
\label{ex:19:inftorus} \label{ex:19:inftorus}
Let $X$ be the infinite torus Let $X$ be the infinite torus

View File

@ -19,6 +19,7 @@
Here I'll try to only use multiplicative notation. Here I'll try to only use multiplicative notation.
\end{remark} \end{remark}
}{} }{}
We will be studying projections to the first $d$ coordinates, We will be studying projections to the first $d$ coordinates,
i.e. i.e.
\[ \[
@ -34,9 +35,9 @@ For $d = 1$ we get the circle rotation $x \mapsto e^{\i \alpha} x$.
\[ \[
H_m \coloneqq \{x \in S^1 : x^m = 0\} H_m \coloneqq \{x \in S^1 : x^m = 0\}
\] \]
for some $m \in \Z$. for some $m \in \Z$.%
\footnote{cf.~\yaref{s12e2}}
\end{fact} \end{fact}
\todo{Homework!}
We will show that $\tau_d$ is minimal for all $d$, We will show that $\tau_d$ is minimal for all $d$,
i.e.~every orbit is dense. i.e.~every orbit is dense.
From this it will follow that $\tau$ is minimal. From this it will follow that $\tau$ is minimal.
@ -45,6 +46,8 @@ Let $\pi_n\colon X \to (S^1)^n$ be the projection to the first $n$
coordinates. coordinates.
% TODO ANKI-MARKER
\begin{lemma} \begin{lemma}
\label{lem:lec20:1} \label{lem:lec20:1}

View File

@ -146,9 +146,7 @@ For this we define
% TODO since for $\overline{x}, \overline{y} \in \mathbb{K}^I$, % TODO since for $\overline{x}, \overline{y} \in \mathbb{K}^I$,
% $d(x_\alpha, y_\alpha) = d((f(\overline{x})_\alpha, (f(\overline{y})_\alpha))$. % $d(x_\alpha, y_\alpha) = d((f(\overline{x})_\alpha, (f(\overline{y})_\alpha))$.
\item Minimality:% \item Minimality:%
\gist{% \notexaminable{%
\footnote{This is not relevant for the exam.}
Let $\langle E_n : n < \omega \rangle$ Let $\langle E_n : n < \omega \rangle$
be an enumeration of a countable basis for $\mathbb{K}^I$. be an enumeration of a countable basis for $\mathbb{K}^I$.
@ -165,11 +163,10 @@ For this we define
is dense in $\overline{x} \mapsto f(\overline{x})$. is dense in $\overline{x} \mapsto f(\overline{x})$.
Since the flow is distal, it suffices to show Since the flow is distal, it suffices to show
that one orbit is dense (cf.~\yaref{thm:distalflowpartition}). that one orbit is dense (cf.~\yaref{thm:distalflowpartition}).
}{ Not relevant for the exam.} }
\item The order of the flow is $\eta$:% \item The order of the flow is $\eta$:%
\gist{% \notexaminable{%
\footnote{This is not relevant for the exam.}
Let $\overline{f} = (f_i)_{i \in I} \in \mathbb{K}_I$. Let $\overline{f} = (f_i)_{i \in I} \in \mathbb{K}_I$.
Consider the flows we get from $(f_i)_{i < j}$ Consider the flows we get from $(f_i)_{i < j}$
resp.~$(f_i)_{i \le j}$ resp.~$(f_i)_{i \le j}$
@ -193,6 +190,6 @@ For this we define
\end{IEEEeqnarray*} \end{IEEEeqnarray*}
Beleznay and Foreman show that this is open and dense.% Beleznay and Foreman show that this is open and dense.%
% TODO similarities to the lemma used today % TODO similarities to the lemma used today
}{ Not relevant for the exam.} }
\end{itemize} \end{itemize}
\end{proof} \end{proof}

View File

@ -37,10 +37,9 @@ Let $I$ be a linear order
S & \coloneqq & \{ x \in \LO(\N) :& x \text{ has a least element},\\ S & \coloneqq & \{ x \in \LO(\N) :& x \text{ has a least element},\\
&&& \text{for any $t$, there is $t \oplus 1$, the successor of $t$.}\} &&& \text{for any $t$, there is $t \oplus 1$, the successor of $t$.}\}
\end{IEEEeqnarray*} \end{IEEEeqnarray*}
\todo{Exercise sheet 12} $S$ is Borel.\footnote{cf.~\yaref{s12e1}}
$S$ is Borel.
We will % TODO ? We will
construct a reduction construct a reduction
\begin{IEEEeqnarray*}{rCl} \begin{IEEEeqnarray*}{rCl}
M \colon S &\longrightarrow & C(\mathbb{K}^\N,\mathbb{K})^\N. %\\ M \colon S &\longrightarrow & C(\mathbb{K}^\N,\mathbb{K})^\N. %\\

View File

@ -1,10 +1,10 @@
\lecture{24}{2024-01-23}{Combinatorics!} \lecture{24}{2024-01-23}{Combinatorics!}
% ANKI 2
\subsection{Applications to Combinatorics} % Ramsey Theory} \subsection{Applications to Combinatorics} % Ramsey Theory}
% TODO Define Ultrafilter
\begin{definition} \begin{definition}
An \vocab{ultrafilter} on $\N$ (or any other set) An \vocab{ultrafilter} on $\N$ (or any other set)
is a family $\cU \subseteq \cP(\N)$ is a family $\cU \subseteq \cP(\N)$
@ -44,6 +44,7 @@
for $\{ n \in \N : \phi(n)\} \in \cU$. for $\{ n \in \N : \phi(n)\} \in \cU$.
We say that $\phi(n)$ holds for \vocab{$\cU$-almost all} $n$. We say that $\phi(n)$ holds for \vocab{$\cU$-almost all} $n$.
\end{notation} \end{notation}
\gist{%
\begin{observe} \begin{observe}
Let $\phi(\cdot )$, $\psi(\cdot )$ be formulas. Let $\phi(\cdot )$, $\psi(\cdot )$ be formulas.
@ -53,6 +54,7 @@
\item $(\cU n) ~\lnot \phi(n) \iff \lnot (\cU n)~ \phi(n)$. \item $(\cU n) ~\lnot \phi(n) \iff \lnot (\cU n)~ \phi(n)$.
\end{enumerate} \end{enumerate}
\end{observe} \end{observe}
}{}
\begin{lemma} \begin{lemma}
\label{lem:ultrafilterlimit} \label{lem:ultrafilterlimit}
Let $X $ be a compact Hausdorff space. Let $X $ be a compact Hausdorff space.
@ -70,7 +72,10 @@
\begin{notation} \begin{notation}
In this case we write $x = \ulim{\cU}_n x_n$. In this case we write $x = \ulim{\cU}_n x_n$.
\end{notation} \end{notation}
\begin{refproof}{lem:ultrafilterlimit}[sketch] \begin{refproof}{lem:ultrafilterlimit}\footnote{The proof from the lecture only works
for metric spaces.}
\gist{
For metric spaces:
Whenever we write $X = Y \cup Z$ Whenever we write $X = Y \cup Z$
we have $(\cU n) x_n \in Y$ we have $(\cU n) x_n \in Y$
or $(\cU n) x_n \in Z$. or $(\cU n) x_n \in Z$.
@ -85,8 +90,13 @@
$C \in \cP_{n+1} \implies \exists C \subseteq D \in \cP_{n}$ $C \in \cP_{n+1} \implies \exists C \subseteq D \in \cP_{n}$
and and
$C_1 \supseteq C_2 \supseteq \ldots$, $C_i \in \cP_i $ $\implies | \bigcap_{i} C_i| = 1$. $C_1 \supseteq C_2 \supseteq \ldots$, $C_i \in \cP_i $ $\implies | \bigcap_{i} C_i| = 1$.
It is clear that we can do this for metric spaces, It is clear that we can do this for metric spaces.
but such partition can be found for compact Hausdorff spaces as well.
}{}
See \yaref{thm:uflimit} for the full proof.
See
\yaref{fact:compactiffufconv} and
\yaref{fact:hdifffilterlimit} for a more general statement.
\end{refproof} \end{refproof}
Let $\beta \N$ be the Čech-Stone compactification of $\N$, Let $\beta \N$ be the Čech-Stone compactification of $\N$,
@ -120,15 +130,14 @@ This gives $+ \colon \beta\N \times \beta\N \to \beta\N$.
This is not commutative, This is not commutative,
but associative and $a \mapsto a + b$ is continuous but associative and $a \mapsto a + b$ is continuous
for a fixed $b$. for a fixed $b$,
This is called a left compact topological semigroup. i.e.~it is a left compact topological semigroup.
Let $X$ be a compact Hausdorff space Let $X$ be a compact Hausdorff space
and let $T \colon X \to X$ be continuous.% and let $T \colon X \to X$ be continuous.%
\footnote{Note that this need not be a homeomorphism, i.e.~we only get a $\N$-action \footnote{Note that this may not be a homeomorphism, i.e.~we only get a $\N$-action
but not a $\Z$-action.} but not a $\Z$-action.}
For any $\cU \in \beta\N$, we define $T^{\cU}$ by For any $\cU \in \beta\N$, we define $T^{\cU}$ by
@ -157,7 +166,6 @@ is not necessarily continuous.
\[ \[
\forall n.~\exists k < M.~ T^{n+k}(x) \in G. \forall n.~\exists k < M.~ T^{n+k}(x) \in G.
\] \]
\end{definition} \end{definition}
\begin{fact} \begin{fact}
Let $\cU, \cV \in \beta\N$ Let $\cU, \cV \in \beta\N$

View File

@ -7,15 +7,17 @@ Let $\beta\N$ denote the set of ultrafilters on $\N$.
where a basis consist of sets where a basis consist of sets
$V_A \coloneqq \{p \in \beta\N : A \in p\}, A \subseteq \N$. $V_A \coloneqq \{p \in \beta\N : A \in p\}, A \subseteq \N$.
\gist{%
(For $A, B \subseteq \N$ we have $V_{A \cap B} = V_{A} \cap V_B$ (For $A, B \subseteq \N$ we have $V_{A \cap B} = V_{A} \cap V_B$
and $\beta\N = V_\N$.) and $\beta\N = V_\N$.)
}{}
\item Note also that for $A, B \subseteq \N$, \item Note also that for $A, B \subseteq \N$,
$V_{A \cup B} = V_A \cup V_B$, $V_{A \cup B} = V_A \cup V_B$,
$V_{A^c} = \beta\N \setminus V_A$. $V_{A^c} = \beta\N \setminus V_A$.
\end{itemize} \end{itemize}
\end{fact} \end{fact}
\gist{%
\begin{observe} \begin{observe}
\label{ob:bNclopenbasis} \label{ob:bNclopenbasis}
Note that the basis is clopen. In particular Note that the basis is clopen. In particular
@ -25,6 +27,7 @@ Let $\beta\N$ denote the set of ultrafilters on $\N$.
If $F$ is closed, then $U = \beta\N \setminus F = \bigcup_{i\in I} V_{A_i}$, If $F$ is closed, then $U = \beta\N \setminus F = \bigcup_{i\in I} V_{A_i}$,
so $F = \bigcap_{i \in I} V_{\N \setminus A_i}$. so $F = \bigcap_{i \in I} V_{\N \setminus A_i}$.
\end{observe} \end{observe}
}{}
\begin{fact} \begin{fact}
\label{fact:bNhd} \label{fact:bNhd}
@ -54,12 +57,14 @@ Let $\beta\N$ denote the set of ultrafilters on $\N$.
$\bigcap_{j=1}^k F_{i_j} \neq \emptyset$. $\bigcap_{j=1}^k F_{i_j} \neq \emptyset$.
We need to show that $\bigcap_{i \in I} F_i \neq \emptyset$. We need to show that $\bigcap_{i \in I} F_i \neq \emptyset$.
\gist{%
Replacing each $F_i$ by $V_{A_j^i}$ such Replacing each $F_i$ by $V_{A_j^i}$ such
that $F_i = \bigcap_{j \in J_i} V_{A_j^i}$ that $F_i = \bigcap_{j \in J_i} V_{A_j^i}$
(cf.~\yaref{ob:bNclopenbasis}) (cf.~\yaref{ob:bNclopenbasis})
we may assume that $F_i$ is of the form $V_{A_i}$. we may assume that $F_i$ is of the form $V_{A_i}$.
We get $\{F_i = V_{A_i} : i \in I\}$ We get $\{F_i = V_{A_i} : i \in I\}$
with the finite intersection property. with the finite intersection property.
}{Wlog.~$F_i = V_{A_i}$.}
Hence Hence
$\{A_i : i \in I\} \mathbin{\text{\reflectbox{$\coloneqq$}}} \cF_0$ $\{A_i : i \in I\} \mathbin{\text{\reflectbox{$\coloneqq$}}} \cF_0$
has the finite intersection property. has the finite intersection property.
@ -78,11 +83,10 @@ Let $\beta\N$ denote the set of ultrafilters on $\N$.
\item $ \{\hat{n}\} $ is open in $\beta\N$ for all $n \in \N$. \item $ \{\hat{n}\} $ is open in $\beta\N$ for all $n \in \N$.
\item $\N \subseteq \beta\N$ is dense. \item $\N \subseteq \beta\N$ is dense.
\end{itemize} \end{itemize}
\todo{Easy exercise}
% TODO write down (exercise)
\end{fact} \end{fact}
\begin{theorem} \begin{theorem}
\label{thm:uflimit}
For every compact Hausdorff space $X$, For every compact Hausdorff space $X$,
a sequence $(x_n)$ in $X$, a sequence $(x_n)$ in $X$,
and $\cU \in \beta\N$, and $\cU \in \beta\N$,
@ -132,6 +136,11 @@ Let $\beta\N$ denote the set of ultrafilters on $\N$.
% TODO general fact: continuous functions agreeing on a dense set % TODO general fact: continuous functions agreeing on a dense set
% agree everywhere (fact section) % agree everywhere (fact section)
\end{proof} \end{proof}
\begin{trivial}+
$\beta$ is a functor from the category of topological
spaces to the category of compact Hausdorff spaces.
It is left adjoint to the inclusion functor.
\end{trivial}
% RECAP % RECAP
\gist{% \gist{%
@ -216,13 +225,12 @@ to obtain
Take $x_2 > x_1$ that satisfies this. Take $x_2 > x_1$ that satisfies this.
\item Suppose we have chosen $\langle x_i : i < n \rangle$. \item Suppose we have chosen $\langle x_i : i < n \rangle$.
Since $\cU$ is idempotent, we have Since $\cU$ is idempotent, we have
\[ \begin{IEEEeqnarray*}{rCl}
(\cU n)[ (\cU n)&& n \in P\\
n \in P &\land& (\cU_k) n + k \in P\\
\land (\cU_k) n + k \in P &\land& \forall {I \subseteq n}.~ (\sum_{i \in I} x_i + n \in P)\\
\land \forall {I \subseteq n}.~ (\sum_{i \in I} x_i + n \in P) &\land& (\cU_k)\left( \forall {I \subseteq n}.~ (\sum_{i \in I} x_i + n + k) \in P\right).
\land (\cU_k)\left( \forall {I \subseteq n}.~ (\sum_{i \in I} x_i + n + k) \in P\right). \end{IEEEeqnarray*}
\]
Chose $x_n > x_{n-1}$ that satisfies this. Chose $x_n > x_{n-1}$ that satisfies this.
\end{itemize} \end{itemize}
Set $H \coloneqq \{x_i : i < \omega\}$. Set $H \coloneqq \{x_i : i < \omega\}$.
@ -231,6 +239,3 @@ to obtain
Next time we'll see another proof of this theorem. Next time we'll see another proof of this theorem.

View File

@ -3,12 +3,36 @@
% Points: 15 / 16 % Points: 15 / 16
\nr 1 \nr 1
\todo{handwritten solution} Let $(X,d)$ be a metric space and $\emptyset \neq A \subseteq X$.
Let $d(x,A) \coloneqq \inf(d(x,a) : a \in A\}$.
\begin{itemize}
\item $d(-,A)$ is uniformly continuous:
Clearly $|d(x,A) - d(y,A)| \le d(x,y)$.
\todo{Add details}
\item $d(x,A) = 0 \iff x \in \overline{A}$.
$d(x,A) = 0$ iff there is a sequence in $A$
converging towards $x$ iff $x \in \overline{A}$.
\end{itemize}
\nr 2 \nr 2
Let $X$ be a discrete space.
For $f,g \in X^{\N}$ define
\[
d(f,g) \coloneqq \begin{cases}
(1 + \min \{n: f(n) \neq g(n)\})^{-1} &: f \neq g,\\
0 &: f= g.
\end{cases}
\]
\begin{enumerate}[(a)] \begin{enumerate}[(a)]
\item $d$ is an ultrametric: \item $d$ is an \vocab{ultrametric},
i.e.~$d(f,g) \le \max \{d(f,h), d(g,h)\}$ for all $f,g,h \in X^{\N}$ :
Let $f,g,h \in X^{\N}$. Let $f,g,h \in X^{\N}$.
@ -70,10 +94,15 @@
\nr 3 \nr 3
Consider $\N$ as a discrete space and $\N^{\N}$ with the product topology.
Let
\[
S_{\infty} = \{f\colon \N \to \N \text{ bijective}\} \subseteq \N^{\N}.
\]
\begin{enumerate}[(a)] \begin{enumerate}[(a)]
\item $S_{\infty}$ is a Polish space: \item $S_{\infty}$ is a Polish space:
From (2) we know that $\N^{\N}$ is Polish. From \yaref{s1e2} we know that $\N^{\N}$ is Polish.
Hence it suffices to show that $S_{\infty}$ is $G_{\delta}$ Hence it suffices to show that $S_{\infty}$ is $G_{\delta}$
with respect to $\N^\N$. with respect to $\N^\N$.
@ -111,69 +140,9 @@
Clearly there cannot exist a finite subcover Clearly there cannot exist a finite subcover
as $B$ is the disjoint union of the $B_j$. as $B$ is the disjoint union of the $B_j$.
% TODO Think about this
\end{enumerate} \end{enumerate}
\nr 4 \nr 4
% (uniform metric)
%
% \begin{enumerate}[(a)]
% \item $d_u$ is a metric on $\cC(X,Y)$:
%
% It is clear that $d_u(f,f) = 0$.
%
% Let $f \neq g$. Then there exists $x \in X$ with
% $f(x) \neq g(x)$, hence $d_u(f,g) \ge d(f(x), g(x)) > 0$.
%
% Since $d$ is symmetric, so is $d_u$.
%
% Let $f,g,h \in \cC(X,Y)$.
% Take some $\epsilon > 0$
% choose $x_1, x_2 \in X$
% with $d_u(f,g) \le d(f(x_1), g(x_1)) + \epsilon$,
% $d_u(g,h) \le d(g(x_2), h(x_2)) + \epsilon$.
%
% Then for all $x \in X$
% \begin{IEEEeqnarray*}{rCl}
% d(f(x), h(x)) &\le &
% d(f(x), g(x)) + d(g(x), h(x))\\
% &\le & d(f(x_1), g(x_1)) + d(g(x_2), h(x_2))-2\epsilon\\
% &\le & d_u(f,g) + d_u(g,h) - 2\epsilon.
% \end{IEEEeqnarray*}
% Thus $d_u(f,g) \le d_u(f,g) + d_u(g,h) - 2\epsilon$.
% Taking $\epsilon \to 0$ yields the triangle inequality.
%
% \item $\cC(X,Y)$ is a Polish space:
% \todo{handwritten solution}
%
% \begin{itemize}
% \item $d_u$ is a complete metric:
%
% Let $(f_n)_n$ be a Cauchy series with respect to $d_u$.
%
% Then clearly $(f_n(x))_n$ is a Cauchy sequence with respect
% to $d$ for every $x$.
% Hence there exists a pointwise limit $f$ of the $f_n$.
% We need to show that $f$ is continuous.
%
% %\todo{something something uniform convergence theorem}
%
% \item $(\cC(X,Y), d_u)$ is separable:
%
% Since $Y$ is separable, there exists a countable
% dense subset $S \subseteq Y$.
%
% Consider $\cC(X,S) \subseteq \cC(X,Y)$.
% Take some $f \in \cC(X,Y)$.
% Since $X$ is compact,
%
%
% % TODO
%
% \end{itemize}
% \end{enumerate}
\begin{fact} \begin{fact}
Let $X $ be a compact Hausdorff space. Let $X $ be a compact Hausdorff space.
Then the following are equivalent: Then the following are equivalent:
@ -205,7 +174,7 @@
Let $X$ be compact Polish\footnote{compact metrisable $\implies$ compact Polish} Let $X$ be compact Polish\footnote{compact metrisable $\implies$ compact Polish}
and $Y $ Polish. and $Y $ Polish.
Let $\cC(X,Y)$ be the set of continuous functions $X \to Y$. Let $\cC(X,Y)$ be the set of continuous functions $X \to Y$.
Consider the metric $d_u(f,g) \coloneqq \sup_{x \in X} |d(f(x), g(x))|$. Consider the \vocab{uniform metric} $d_u(f,g) \coloneqq \sup_{x \in X} |d(f(x), g(x))|$.
Clearly $d_u$ is a metric. Clearly $d_u$ is a metric.
\begin{claim} \begin{claim}
@ -243,7 +212,7 @@ Clearly $d_u$ is a metric.
for each $y \in X_m$. for each $y \in X_m$.
Then $\bigcup_{m,n} D_{m,n}$ is dense in $\cC(X,Y)$: Then $\bigcup_{m,n} D_{m,n}$ is dense in $\cC(X,Y)$:
Indeed if $f \in \cC(X,Y)$ and $\eta > 0$, Indeed if $f \in \cC(X,Y)$ and $\eta > 0$,
we finde $n > \frac{3}{\eta}$ and $m$ such that $f \in C_{m,n}$, we find $n > \frac{3}{\eta}$ and $m$ such that $f \in C_{m,n}$,
since $f$ is uniformly continuous. since $f$ is uniformly continuous.
Let $g \in D_{m,n}$ be such that $\forall y \in X_m.~d(f(y), g(y)) < \frac{1}{n+1}$. Let $g \in D_{m,n}$ be such that $\forall y \in X_m.~d(f(y), g(y)) < \frac{1}{n+1}$.
We have $d_u(f,g) \le \eta$, We have $d_u(f,g) \le \eta$,

View File

@ -12,6 +12,14 @@
\nr 1 \nr 1
Let $X$ be a Polish space.
Then there exists an injection $f\colon X \to 2^\omega$
such that for each $n < \omega$,
the set $f^{-1}(\{(y_n) \in 2^\omega : y_n = 1\})$
is open.
Moreover if $V \subseteq 2^{ \omega}$ is closed,
then $f^{-1}(V)$ is $G_\delta$.
Let $(U_i)_{i < \omega}$ be a countable base of $X$. Let $(U_i)_{i < \omega}$ be a countable base of $X$.
Define Define
\begin{IEEEeqnarray*}{rCl} \begin{IEEEeqnarray*}{rCl}
@ -19,6 +27,7 @@ Define
x &\longmapsto & (x_i)_{i < \omega} x &\longmapsto & (x_i)_{i < \omega}
\end{IEEEeqnarray*} \end{IEEEeqnarray*}
where $x_i = 1$ iff $x \in U_i$ and $x_i = 0$ otherwise. where $x_i = 1$ iff $x \in U_i$ and $x_i = 0$ otherwise.
\gist{
Then $f^{-1}(\{y = (y_n) \in 2^\omega | y_n = 1\}) = U_n$ Then $f^{-1}(\{y = (y_n) \in 2^\omega | y_n = 1\}) = U_n$
is open. is open.
We have that $f$ is injective since $X$ is T1. We have that $f$ is injective since $X$ is T1.
@ -51,17 +60,21 @@ Since $2^{n} \setminus \left( \prod_{i < n} X_i \right)$
is finite, we get that is finite, we get that
$f^{-1}(2^{\omega} \setminus ((\prod_{i <n} X_{i}) \times 2^{\omega}))$ $f^{-1}(2^{\omega} \setminus ((\prod_{i <n} X_{i}) \times 2^{\omega}))$
is $G_\delta$ as a finite union of $G_{\delta}$ sets. is $G_\delta$ as a finite union of $G_{\delta}$ sets.
}{}
\nr 2 \nr 2
Let $X$ be a Polish space. Then $X$ is homeomorphic to a closed subspace of $\R^{ \omega}$ :
\todo{handwritten solution} \todo{handwritten solution}
(b) % \begin{itemize}
Let $f(x^{(i)})$ be a sequence in $f(X)$. % \item
Suppose that $f(x^{(i)}) \to y$. % Let $f(x^{(i)})$ be a sequence in $f(X)$.
We have that $f^{-1} = \pi_{\text{odd}}$ is continuous. % Suppose that $f(x^{(i)}) \to y$.
Then $\pi_{\text{odd}}(f(x^{(i)}) \to \pi_{\text{odd}}(y)$. % We have that $f^{-1} = \pi_{\text{odd}}$ is continuous.
Since $\pi_{\text{even}}$ converges, we have $\pi_{\text{odd}}(y) \in X$. % Then $\pi_{\text{odd}}(f(x^{(i)}) \to \pi_{\text{odd}}(y)$.
% Since $\pi_{\text{even}}$ converges, we have $\pi_{\text{odd}}(y) \in X$.
% \end{itemize}
\nr 3 \nr 3
@ -130,6 +143,13 @@ Since $\pi_{\text{even}}$ converges, we have $\pi_{\text{odd}}(y) \in X$.
\end{proof} \end{proof}
\nr 4 \nr 4
Define
\begin{IEEEeqnarray*}{rCl}
f\colon \omega^{\omega} &\longrightarrow & 2^\omega \\
(x_n)&\longmapsto & 0^{x_0} 1 0^{x_1} 1 \ldots.
\end{IEEEeqnarray*}
\begin{enumerate}[(1)] \begin{enumerate}[(1)]
\item $f$ is a topological embedding: \item $f$ is a topological embedding:
Consider a basic open set Consider a basic open set

View File

@ -8,7 +8,7 @@ Let $A \neq \emptyset$ be discrete.
For $D \subseteq A^{\omega}$, For $D \subseteq A^{\omega}$,
let let
\[ \[
T_D \coloneqq \{x\defon{n} \in A^{<\omega} | x \in D, n \in \N\}.. T_D \coloneqq \{x\defon{n} \in A^{<\omega} | x \in D, n \in \N\}.
\] \]
\begin{enumerate}[(a)] \begin{enumerate}[(a)]
\item For any $D \subseteq A^\omega$, $T_D$ is a pruned tree: \item For any $D \subseteq A^\omega$, $T_D$ is a pruned tree:

View File

@ -267,14 +267,14 @@ $X_\omega \coloneqq \bigcap X_i$ and $Y_\omega \coloneqq \bigcap Y_i$.
% https://q.uiver.app/#q=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 % https://q.uiver.app/#q=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
\adjustbox{scale=0.7,center}{% \adjustbox{scale=0.7,center}{%
\[\begin{tikzcd} \begin{tikzcd}
{X \setminus X_\omega =} & {(X_0 \setminus X_1)} & \cup & {(X_1 \setminus X_2)} & \cup & {(X_2 \setminus X_3)} & \cdots & {} \\ {X \setminus X_\omega =} & {(X_0 \setminus X_1)} & \cup & {(X_1 \setminus X_2)} & \cup & {(X_2 \setminus X_3)} & \cdots & {} \\
{Y\setminus Y_\omega =} & {(Y_0 \setminus Y_1)} & \cup & {(Y_1 \setminus Y_2)} & \cup & {(Y_2 \setminus Y_3)} & \cdots & {} {Y\setminus Y_\omega =} & {(Y_0 \setminus Y_1)} & \cup & {(Y_1 \setminus Y_2)} & \cup & {(Y_2 \setminus Y_3)} & \cdots & {}
\arrow["f"'{pos=0.7}, from=1-2, to=2-4] \arrow["f"'{pos=0.7}, from=1-2, to=2-4]
\arrow["g"{pos=0.1}, from=2-2, to=1-4] \arrow["g"{pos=0.1}, from=2-2, to=1-4]
\arrow["f"{pos=0.8}, from=1-6, to=2-8] \arrow["f"{pos=0.8}, from=1-6, to=2-8]
\arrow["g"{pos=0.1}, from=2-6, to=1-8] \arrow["g"{pos=0.1}, from=2-6, to=1-8]
\end{tikzcd}\] \end{tikzcd}
} }
By \autoref{thm:lusinsouslin} By \autoref{thm:lusinsouslin}

View File

@ -63,7 +63,7 @@ Flows are always on non-empty spaces $X$.
\begin{proof} \begin{proof}
(i) $\implies$ (ii): (i) $\implies$ (ii):
Let $(Y,T)$ be a subflow of $(X,T)$. Let $(Y,T)$ be a subflow of $(X,T)$.
take $y \in Y$. Then $Ty$ is dense in mKX. take $y \in Y$. Then $Ty$ is dense in $X$.
But $Ty \subseteq Y$, so $Y$ is dense in $X$. But $Ty \subseteq Y$, so $Y$ is dense in $X$.
Since $Y$ is closed, we get $Y = X$. Since $Y$ is closed, we get $Y = X$.

View File

@ -4,9 +4,58 @@
\nr 1 \nr 1
% Examinable % Examinable
% TODO (there is a more direct way to do it, not using analytic / coanalytic) Let $\LO(\N) \overset{\text{closed}}{\subseteq} 2^{\N\times \N}$ denote the set of linear orders on $\N$.
Let $S \subseteq \LO(\N)$ be the set of orders having a least
element and such that every element has an immediate successor.
\begin{itemize}
\item $S$ is Borel in $\LO(\N)$:
Let $M_n \subseteq \LO(\N)$ be the set of orders with minimal element $n$.
Let $I_{n,m} \subseteq \LO(\N)$ be the set of orders such
that $m$ is the immediate successor of $n$.
Clearly $S = \left(\bigcap_n \bigcup_{m\neq n} I_{n,m}\right) \cap \bigcup_n M_n$,
so it suffices to show that $M_n$ and $I_{n,m}$ are Borel.
It is $M_n = \bigcap_{m\neq n} \{\prec : m \not\prec n\}$
and $I_{n,m} = \{\prec: n \prec m\} \cap \bigcap_{i} \{\prec : n \preceq i \preceq m \implies n = i \lor n = m \}$.
\item Give an example of an element of $S$ which is not well-ordered:
Consider $\{1 - \frac{1}{n} : n \in \N^+\} \cup \{1 + \frac{1}{n} : n \in \N^{+}\} \subseteq \R$
with the order $<_\R$.
This is an element of $S$,
but $\{x \in S: x \ge 1\}$ has no minimal element,
hence it is not well-ordered.
\end{itemize}
\nr 2 \nr 2
% Examinable % Examinable
Recall the definition of the circle shift flow $(\R / \Z, \Z)$
with parameter $\alpha \in \R$, $1 \cdot x \coloneqq x + \alpha$.
\begin{itemize}
\item If $\alpha \not\in \Q$, then $(\R / \Z, \Z)$ is minimal:
This is known as \href{https://en.wikipedia.org/wiki/Dirichlet's_approximation_theorem}{Dirichlet's Approximation Theorem}.
\item Consider $\R/\Z$ as a topological group.
Any subgroup $H$ of $\R / \Z$ is dense in $\R / \Z$
or of the form $H = \{ x \in \R / \Z | mx = 0\}$
for some $m \in \Z$.
If $H$ contains an irrational element $\alpha$, then
it is dense by the previous point.
Suppose that $H \subseteq \Q / \Z$.
Let $D$ be the set of denominators of elements of $H$
written as irreducible fractions.
If $D$ is finite,
then $H = \{x \in \R / \Z : \mathop{lcm}(D)x = 0\}$.
Otherwise $H$ is dense, as it contains
elements of arbitrarily large denominator.
\end{itemize}
\nr 3 \nr 3
@ -35,11 +84,12 @@
\nr 4 \nr 4
% Examinable! % Examinable!
% TODO THINK!
\gist{%
% RECAP % RECAP
Let $X$ be a metrizable topological space. Let $X$ be a metrizable topological space
and let $K(X) \coloneqq \{ K \subseteq X : K \text{ compact}\}$.
Let $K(X) \coloneqq \{ K \subseteq X : \text{ compact}\}$.
The Vietoris topology has a basis given by The Vietoris topology has a basis given by
$\{K \subseteq U\}$, $U$ open (type 1) $\{K \subseteq U\}$, $U$ open (type 1)
@ -54,19 +104,21 @@ $\max_{a \in A} d(a,B)$.
On previous sheets, we checked that $d_H$ is a metric. On previous sheets, we checked that $d_H$ is a metric.
If $X$ is separable, then so is $K(X)$. If $X$ is separable, then so is $K(X)$.
% END RECAP % END RECAP
}{}
\begin{fact} \begin{fact}
\label{fact:s12e4}
Let $(X,d)$ be a complete metric space. Let $(X,d)$ be a complete metric space.
Then so is $(K(X), d_H)$. Then so is $(K(X), d_H)$.
\end{fact} \end{fact}
\begin{proof} \begin{refproof}{fact:s12e4}
We need to show that $(K(X), d_H)$ is complete. We need to show that $(K(X), d_H)$ is complete.
Let $(K_n)_{ n< \omega}$ be Cauchy in $(K(X), d_H)$. Let $(K_n)_{ n< \omega}$ be Cauchy in $(K(X), d_H)$.
Wlog.~$K_n \neq \emptyset$ for all $n$. Wlog.~$K_n \neq \emptyset$ for all $n$.
Let $K = \{ x \in X : \forall x \in U \overset{\text{open}}{\subseteq} X.~ Let $K = \{ x \in X : \forall x \in U \overset{\text{open}}{\subseteq} X.~
\text{ $X$ intersects $K_n$ for infinitely many $n$}\}$. \text{ $U \cap K_n \neq \emptyset$ for infinitely many $n$}\}$.
Equivalently, Equivalently,
$K = \{x : x \text{ is a cluster point of some subsequence $(x_n)$ with $x_n \in K_n$ for all $K_n$}\}$. $K = \{x : x \text{ is a cluster point of some subsequence $(x_n)$ with $x_n \in K_n$ for all $K_n$}\}$.
@ -74,12 +126,12 @@ Then so is $(K(X), d_H)$.
(A cluster point is a limit of some subsequence). (A cluster point is a limit of some subsequence).
\begin{claim} \begin{claim}
\label{fact:s12e4:c1}
$K_n \to K$. $K_n \to K$.
\end{claim} \end{claim}
\begin{subproof} \begin{refproof}{fact:s12e4:c1}
Note that $K$ is closed (the complement is open). Note that $K$ is closed (the complement is open).
\begin{claim} \begin{claim}
$K \neq \emptyset$. $K \neq \emptyset$.
\end{claim} \end{claim}
@ -110,7 +162,7 @@ Then so is $(K(X), d_H)$.
space, it is complete. space, it is complete.
So it suffices to show that $K$ is totally bounded. So it suffices to show that $K$ is totally bounded.
Let $\epsilon > 0$ Let $\epsilon > 0$.
Take $N$ such that $d_H(K_i,K_j) < \epsilon$ Take $N$ such that $d_H(K_i,K_j) < \epsilon$
for all $i,j \ge N$. for all $i,j \ge N$.
@ -151,9 +203,8 @@ Then so is $(K(X), d_H)$.
To do this, construct a sequence of $y_{n_i} \in K_{n_i}$ To do this, construct a sequence of $y_{n_i} \in K_{n_i}$
starting with $y$ such that $d(y_{n_i}, y_{n_{i+1}}) < \frac{\epsilon}{2^{i+2}}$. starting with $y$ such that $d(y_{n_i}, y_{n_{i+1}}) < \frac{\epsilon}{2^{i+2}}$.
(same trick as before). (same trick as before).
\end{subproof} \end{refproof}
\end{refproof}
\end{proof}
\begin{fact} \begin{fact}
If $X$ is compact metrisable, If $X$ is compact metrisable,
@ -174,9 +225,3 @@ Then so is $(K(X), d_H)$.
% TODO complete and totally bounded Sutherland metric and topological spaces % TODO complete and totally bounded Sutherland metric and topological spaces

View File

@ -2,7 +2,7 @@
\tutorial{15}{2024-01-31}{Additions} \tutorial{15}{2024-01-31}{Additions}
The following is not relevant for the exam, The following is not relevant for the exam,
but gives a more general picture. but aims to give a more general picture.
Let $X$ be a topological space Let $X$ be a topological space
and let $\cF$ be a filter on $ X$. and let $\cF$ be a filter on $ X$.
@ -12,6 +12,7 @@ all sets containing an open neighbourhood of $x$,
is contained in $\cF$. is contained in $\cF$.
\begin{fact} \begin{fact}
\label{fact:hdifffilterlimit}
$X$ is Hausdorff iff every filter has at most one limit point. $X$ is Hausdorff iff every filter has at most one limit point.
\end{fact} \end{fact}
\begin{proof} \begin{proof}
@ -21,6 +22,7 @@ is contained in $\cF$.
\end{proof} \end{proof}
\begin{fact} \begin{fact}
\label{fact:compactiffufconv}
$X$ is (quasi-) compact $X$ is (quasi-) compact
iff every ultrafilter converges. iff every ultrafilter converges.
\end{fact} \end{fact}
@ -29,7 +31,7 @@ is contained in $\cF$.
Let $\cU$ be an ultrafilter. Let $\cU$ be an ultrafilter.
Consider the family $\cV = \{\overline{A} : A \in \cU\}$ Consider the family $\cV = \{\overline{A} : A \in \cU\}$
of closed sets. of closed sets.
By the FIP we geht that there exist By the FIP we get that there exist
$c \in X$ such that $c \in \overline{A}$ for all $A \in \cU$. $c \in X$ such that $c \in \overline{A}$ for all $A \in \cU$.
Let $N$ be an open neighbourhood of $c$. Let $N$ be an open neighbourhood of $c$.
If $N^c \in \cU$, then $c \in N^c \lightning$ If $N^c \in \cU$, then $c \in N^c \lightning$
@ -69,17 +71,19 @@ so is $f(\cB)$.
\end{fact} \end{fact}
\begin{proof} \begin{proof}
Consider $(f,g)^{-1}(\Delta) \supseteq A$. Consider $(f,g)^{-1}(\Delta) \supseteq A$.
The RHS is a dense closed set, i.e.~the entire space.
\end{proof} \end{proof}
We can uniquely extend $f\colon X \to Y$ continuous We can uniquely extend a continuous $f\colon X \to Y$
to a continuous $\overline{f}\colon \beta X \to Y$ to a continuous $\overline{f}\colon \beta X \to Y$
by setting $\overline{f}(\cU) \coloneqq \lim_\cU f$. by setting $\overline{f}(\cU) \coloneqq \lim_\cU f$.
Let $V$ be an open neighbourhood of $Y$ in $\overline{f}\left( U) \right) $. % Let $V$ be an open neighbourhood of $y \in \overline{f}\left( U \right)$.
Consider $f^{-1}(V)$. % Consider $f^{-1}(V)$.
Consider the basic open set % Then
\[ % \[
\{\cF \in \beta\N : \cF \ni f^{-1}(V)\}. % \{\cF \in \beta\N : \cF \ni f^{-1}(V)\}
\] % \]
% is a basic open set.
\todo{I missed the last 5 minutes} \todo{I missed the last 5 minutes}